Mach-Zehnder Interferometer interpretation

In summary, the individual is seeking help with interpreting an experiment involving photons and their states and paths. They present their ideas and assumptions, but are corrected by another user who explains the need to properly account for both the direction and path of the photons in a larger, four-dimensional space. The individual then asks for assistance with performing a transformation on one of the paths and for clarification on which assumptions were faulty. They end by expressing their gratitude for any assistance.
  • #1
WWCY
479
12
HW Template missing as it was moved from another forum
Hi all,

I have a problem trying to interpret the mathematics of this experiment and would like some help. I think it's best I write out all my ideas (and misconceptions) so that I can be corrected. An illustration of the setup is below. Thanks in advance for any assistance!

Let the following vectors represent "right-moving" and "upwards-moving" states of photons respectively
$$|r> \ = (1,0)$$
$$|u> \ = (0,1)$$

A beam of photons purely in the "right" state enters the first beam-splitter (represented by the matrix) and exits in a mix of "r" and "u" states
$$\frac{1}{\sqrt{2}}
\left( \begin{array}{cc}
1 & 1 \\
-1 & 1
\end{array} \right)
%
\left( \begin{array}{cc}
1 \\
0
\end{array} \right)
= \frac{1}{\sqrt{2}} \big ( (1,0) + (0,-1) \big)
$$

Assumption 1: ##\frac{1}{\sqrt{2}}(1,0)## represents the beam taking the right-wards path, and ##\frac{1}{\sqrt{2}}(0,-1)## is the beam moving upwards.

In the setup shown, both beams under go a "state-flip" as they hit the mirrors, and change direction and fly towards the second beam-splitter.

However, just for fun, I only want the "u" beam to hit a mirror, while the "r" beam just keeps flying right.

Assumption 2: I can do this by performing the "flip" on the "u" beam only. The "flipper" is represented by the matrix and the resulting state is
$$
\left( \begin{array}{cc}
0 & 1 \\
-1 & 0
\end{array} \right)
%
\left( \begin{array}{cc}
0 \\
-\frac{1}{\sqrt{2}}
\end{array} \right)
= -\frac{1}{\sqrt{2}} (1,0)
$$

Assumption 3: Because both my beams are now "r" states, the probability of finding them as "r" states should be a sweet 1. And thus, summing the two "r-wards" vectors should give me ##(1,0)##. But
$$\frac{1}{\sqrt{2}} \big[ (1,0) + (-1,0) \big] = 0$$
Assumption 4: I could remove the negative sign from the "u" state vector, since the sign plays no physical significance. But this gives me ##\frac{1}{\sqrt{2}}(2,0)## which does not make sense either.

Assistance is greatly appreciated!

Screen Shot 2018-02-07 at 11.33.28 PM.png
 

Attachments

  • Screen Shot 2018-02-07 at 11.33.28 PM.png
    Screen Shot 2018-02-07 at 11.33.28 PM.png
    9.4 KB · Views: 505
  • Like
Likes Douglas Sunday
Physics news on Phys.org
  • #2
First, if you want to compute a probability or generally "weigh" your states, you have to square amplitudes before adding them. Second, when the qubit your are focusing on (the direction) is part of a system with a second qubit (the position), you have to properly trace out the other qubit before adding. Otherwise you'll have problems.

For example, the state ##|0\rangle|0\rangle## looks fine, but apply a Hadamard gate to the second qubit and you get ##|0\rangle (\frac{1}{\sqrt{2}} |0\rangle + \frac{1}{\sqrt{2}} |1\rangle)##. If you simply ignore the second qubit, by replacing its kets with 1s, you'd get ##|0>(\frac{1}{\sqrt{2}} \cdot 1 + \frac{1}{\sqrt{2}} \cdot 1) = |0\rangle\cdot\sqrt{2}##. But that's nonsensical. You can't just ignore the second qubit by replacing its kets with the scalar 1! This is exactly what's happening in your example: the first qubit is the direction of the photon and the second qubit is the position of the photon.

To correctly remove the second qubit from the system you have to "trace it out". Convert your kets into density matrices, and http://www.thphy.uni-duesseldorf.de/~ls3/teaching/1515-QOQI/Additional/partial_trace.pdf. The resulting single-qubit density matrix tells you about the first qubit.
 
  • #3
Strilanc said:
First, if you want to compute a probability or generally "weigh" your states, you have to square amplitudes before adding them. Second, when the qubit your are focusing on (the direction) is part of a system with a second qubit (the position), you have to properly trace out the other qubit before adding. Otherwise you'll have problems.

For example, the state ##|0\rangle|0\rangle## looks fine, but apply a Hadamard gate to the second qubit and you get ##|0\rangle (\frac{1}{\sqrt{2}} |0\rangle + \frac{1}{\sqrt{2}} |1\rangle)##. If you simply ignore the second qubit, by replacing its kets with 1s, you'd get ##|0>(\frac{1}{\sqrt{2}} \cdot 1 + \frac{1}{\sqrt{2}} \cdot 1) = |0\rangle\cdot\sqrt{2}##. But that's nonsensical. You can't just ignore the second qubit by replacing its kets with the scalar 1! This is exactly what's happening in your example: the first qubit is the direction of the photon and the second qubit is the position of the photon.

To correctly remove the second qubit from the system you have to "trace it out". Convert your kets into density matrices, and http://www.thphy.uni-duesseldorf.de/~ls3/teaching/1515-QOQI/Additional/partial_trace.pdf. The resulting single-qubit density matrix tells you about the first qubit.
Hi @Strilanc , thank you for your reply!

Unfortunately I have really only just learned this formalism and I don't understand your post. If you don't mind, could you dumb it down for me?

Perhaps I should have mentioned that I only know stuff like representing states as vectors, transformations as matrices, and the idea that two-state systems can be described by this formalism. Apologies!
 
  • #4
You don't have two photons in the state ##|r\rangle##. You have one photon in the state ##|r\rangle \otimes |\text{top path}\rangle## and one photon in the state ##|r\rangle \otimes |\text{bottom path}\rangle##. Your vector space needs to be four dimensional, not two dimensional, to account for both the direction and the path. In the larger four dimensional space, you will find that the two vectors no longer add up to zero.
 
  • #5
Strilanc said:
You don't have two photons in the state ##|r\rangle##. You have one photon in the state ##|r\rangle \otimes |\text{top path}\rangle## and one photon in the state ##|r\rangle \otimes |\text{bottom path}\rangle##. Your vector space needs to be four dimensional, not two dimensional, to account for both the direction and the path. In the larger four dimensional space, you will find that the two vectors no longer add up to zero.

So let's say I wish to perform some transformation on one of the paths, without performing it on the other, how should I go about doing it?

Also, may I know which one of my assumptions were faulty and how I should have approached them instead?

Thank you for your patience.
 
  • #6
Can anyone assist? Many thanks in advance.
 
  • #7
Bump.
 
  • #8
Hi all, I'd be grateful for some assistance regarding these concepts.

Either that or some advice regarding how I should restructure my post will be greatly appreciated!
 

Related to Mach-Zehnder Interferometer interpretation

1. What is a Mach-Zehnder Interferometer?

A Mach-Zehnder Interferometer is a type of optical instrument that uses the principle of interference to measure small changes in the phase or amplitude of a light wave. It consists of two beamsplitters and two mirrors arranged in a specific configuration, which allows for the splitting, recombining, and interference of two beams of light.

2. How does a Mach-Zehnder Interferometer work?

In a Mach-Zehnder Interferometer, a single beam of light is split into two paths by a beamsplitter. These two beams then reflect off of two separate mirrors before being recombined by another beamsplitter. The resulting interference pattern is then observed and can be used to measure small changes in the phase or amplitude of the light waves.

3. What is the purpose of a Mach-Zehnder Interferometer?

The main purpose of a Mach-Zehnder Interferometer is to measure small changes in the phase or amplitude of light waves. It is commonly used in scientific research and various industries, such as telecommunications, to detect and measure these changes with high precision.

4. What are some potential applications of a Mach-Zehnder Interferometer?

Mach-Zehnder Interferometers have a wide range of applications, including optical communications, interferometric microscopy, and sensing of physical quantities such as temperature, pressure, and strain. They are also used in quantum computing and other advanced technologies.

5. Are there any limitations or challenges in interpreting data from a Mach-Zehnder Interferometer?

One limitation of a Mach-Zehnder Interferometer is that it requires careful alignment and calibration to ensure accurate measurements. Additionally, the interference pattern can be affected by external factors such as vibrations and temperature changes. Therefore, careful data analysis and error correction techniques may be necessary for accurate interpretation of the data.

Similar threads

  • Introductory Physics Homework Help
Replies
8
Views
609
  • Introductory Physics Homework Help
Replies
30
Views
455
  • Introductory Physics Homework Help
Replies
7
Views
237
  • Introductory Physics Homework Help
Replies
10
Views
291
  • Quantum Physics
Replies
13
Views
676
  • Introductory Physics Homework Help
Replies
34
Views
727
  • Introductory Physics Homework Help
Replies
13
Views
1K
  • Introductory Physics Homework Help
Replies
28
Views
398
  • Introductory Physics Homework Help
Replies
6
Views
258
  • Introductory Physics Homework Help
Replies
2
Views
252
Back
Top